K
Khách

Hãy nhập câu hỏi của bạn vào đây, nếu là tài khoản VIP, bạn sẽ được ưu tiên trả lời.

14 tháng 6 2019

\(A+\frac{1}{4}=x+\frac{1}{2}.2\sqrt{x}+\left(\frac{1}{2}\right)^2=\left(\sqrt{x}+\frac{1}{2}\right)^2\ge\left(0+\frac{1}{2}\right)^2=\frac{1}{4}\)

nên: \(A_{min}=0\).Dấu "=" xảy ra khi: \(x=0\)

9 tháng 1 2021

Áp dụng bất đẳng thức Cauchy-Schwarz dạng phân thức, ta được: \(VT=\frac{a^4}{a^2+a^2b-a^3}+\frac{b^4}{b^2+b^2c-b^3}+\frac{c^4}{c^2+c^2a-c^3}\)\(\ge\frac{\left(a^2+b^2+c^2\right)^2}{\left(a^2+b^2+c^2\right)+\left(a^2b+b^2c+c^2a\right)-\left(a^3+b^3+c^3\right)}\)        \(=\frac{1}{1+\left(a^2b+b^2c+c^2a\right)-\left(a^3+b^3+c^3\right)}\)

Ta cần chứng minh \(\frac{1}{1+\left(a^2b+b^2c+c^2a\right)-\left(a^3+b^3+c^3\right)}\ge1\)hay \(a^3+b^3+c^3\ge a^2b+b^2c+c^2a\)

Đây là bất đẳng thức quen thuộc có nhiều cách chứng minh:

** Cách 1: Áp dụng AM - GM, ta được: \(a^3+a^3+b^3\ge3a^2b\)\(b^3+b^3+c^3\ge3b^2c\)\(c^3+c^3+a^3\ge3c^2a\)

Cộng từng vế ba bất đẳng thức trên

** Cách 2: Giả sử \(a\le b\le c\)

Có: \(a^3+b^3+c^3=a^2b+b^2c+c^2a+\left(c^2-a^2\right)\left(b-a\right)+\left(c^2-b^2\right)\left(c-b\right)\ge a^2b+b^2c+c^2a\)

Vậy bất đẳng thức được chứng minh

Đẳng thức xảy ra khi \(a=b=c=\frac{1}{\sqrt{3}}\).

9 tháng 1 2021

Or the following SOS: 

* Hoặc mạnh hơn với a,b,c thực thỏa mãn \(a+b\ge0,b+c\ge0,c+a\ge0\)

\(a^3+b^3+c^3-a^2b-b^2c-c^2a\)

                                            \(=\frac{\left(a^2+b^2-2c^2\right)^2+3\left(a^2-b^2\right)^2+\Sigma_{cyc}4\left(a+b\right)\left(c+a\right)\left(a-b\right)^2}{8\left(a+b+c\right)}\ge0\)

14 tháng 1 2021

Câu đề HN vừa thi hôm trước, sửa thành tìm max

Áp dụng BĐT Bunyakovsky ta có:

\(\left(\sqrt{a+b}+\sqrt{b+c}+\sqrt{c+a}\right)^2\le\left(1^2+1^2+1^2\right)\left(a+b+b+c+c+a\right)\)

\(=6\left(a+b+c\right)\le6\) 

\(\Rightarrow\left(\sqrt{a+b}+\sqrt{b+c}+\sqrt{c+a}\right)\le\sqrt{6}\)

Dấu "=" xảy ra khi a = b = c = 1/3

Làm xong mới thấy không giống lắm hihi:D

10 tháng 1 2021

Xét \(a+b\ge1\Leftrightarrow b\ge1-a\)

Xét \(Q\ge\dfrac{8a^2+1-a}{4a}+\left(1-a\right)^2=\dfrac{8a^2}{4a}+\dfrac{1}{4a}-\dfrac{a}{4a}+1-2a+a^2\)

        \(=2a+\dfrac{1}{4a}-\dfrac{1}{4}+1-2a+a^2\)\(=a^2+\dfrac{1}{4a}+\dfrac{3}{4}\)\(=\left(a^2+\dfrac{1}{8a}+\dfrac{1}{8a}\right)+\dfrac{3}{4}\)

Áp dụng Cosi được \(Q\ge3\sqrt[3]{a^2\cdot\dfrac{1}{8a}\cdot\dfrac{1}{8a}}+\dfrac{3}{4}\)\(=3\sqrt[3]{\dfrac{1}{64}}+\dfrac{3}{4}=\dfrac{3}{4}+\dfrac{3}{4}=\dfrac{3}{2}\) 

Vậy \(Qmin=\dfrac{3}{2}\) khi \(a=b=\dfrac{1}{2}\)

24 tháng 12 2018

\(\frac{8a^2+b}{4a}+b^2=2a+\frac{b}{4a}+b^2=a+a+\frac{b}{4a}+b^2\)

\(\ge a+1-b+\frac{1-a}{4a}+b^2=a+1-b+\frac{1}{4a}-\frac{1}{4}+b^2\)(do \(a+b\ge1\))

\(=\left(a+\frac{1}{4a}\right)+b^2-b+\frac{1}{4}+\frac{1}{2}\)

\(\ge2\sqrt{a\cdot\frac{1}{4a}}+\left(b-\frac{1}{2}\right)^2+\frac{1}{2}\)

\(\ge2\cdot\frac{1}{2}+\frac{1}{2}=\frac{3}{2}\)

Dấu = khi \(a=b=\frac{1}{2}\)

12 tháng 7 2016

a) ĐKXĐ: x\(\ne\) 0;4

Ta có: Q= \(\left(\frac{4\sqrt{x}}{2+\sqrt{x}}+\frac{8x}{\left(2-\sqrt{x}\right)\left(2+\sqrt{x}\right)}\right):\left(\frac{\sqrt{x}-1}{\sqrt{x}\left(\sqrt{x}-2\right)}-\frac{2}{\sqrt{x}}\right)\)

                   = \(\frac{4\sqrt{x}\cdot\left(2-\sqrt{x}\right)+8x}{\left(2-\sqrt{x}\right)\left(2+\sqrt{x}\right)}:\frac{\sqrt{x}-1-2\cdot\left(\sqrt{x}-2\right)}{\sqrt{x}\left(\sqrt{x}-2\right)}\)

                   =\(\frac{8\sqrt{x}+4x}{\left(2-\sqrt{x}\right)\left(2+\sqrt{x}\right)}\cdot\frac{\sqrt{x}\left(\sqrt{x}-2\right)}{3-\sqrt{x}}\)\(\frac{4\sqrt{x}\cdot\left(2+\sqrt{x}\right)}{2+\sqrt{x}}\cdot\frac{-\sqrt{x}}{3-\sqrt{x}}\)=\(\frac{-4}{3-\sqrt{x}}\)=\(\frac{4}{\sqrt{x}-3}\)

b) Q=-1 => \(\frac{4}{\sqrt{x}-3}=-1\)

<=> \(4=3-\sqrt{x}\)

<=> \(\sqrt{x}=-1\) (vô lí)

Vậy ko tìm được x.

12 tháng 7 2016

kamsamita vui

22 tháng 7 2019

Em làm thử nhé!

Bài 1: \(A=\left[\frac{a^2}{b-1}+4\left(b-1\right)\right]+\left[\frac{b^2}{a-1}+4\left(a-1\right)\right]-4\left(a+b\right)+8\)

Cauchy vào là ra rồi ạ;)

Bài 2: Em chịu

22 tháng 7 2019

2) Có: \(\sqrt{ab}\le\frac{a+b}{2}=1\)\(\sqrt{a}+\sqrt{b}=\sqrt{\left(\sqrt{a}+\sqrt{b}\right)^2}\le\sqrt{2\left(a+b\right)}=2\)

\(\frac{a}{\sqrt{b}}+\frac{b}{\sqrt{a}}=\frac{\left(\sqrt{a}\right)^3+\left(\sqrt{b}\right)^3}{\sqrt{ab}}\ge\left(\sqrt{a}\right)^3+\left(\sqrt{b}\right)^3=\frac{a^2}{\sqrt{a}}+\frac{b^2}{\sqrt{b}}\)

\(\ge\frac{\left(a+b\right)^2}{\sqrt{a}+\sqrt{b}}\ge=\frac{2^2}{2}=2\)

Dấu "=" xảy ra \(\Leftrightarrow\)\(a=b=1\)